Question on Mean Value Theorem

In summary, the equation given for f'(x) allows for the existence of a point (at least one point) ##c## such that f'(c)=0.
  • #1
Titan97
Gold Member
450
18

Homework Statement


Let ###f## be double differentiable function such that ##|f''(x)|\le 1## for all ##x\in [0,1]##. If f(0)=f(1), then,
A)##|f(x)|>1##
B)##|f(x)|<1##
C)##|f'(x)|>1##
D)##|f'(x)|<1##

Homework Equations


MVT: $$f'(c)=\frac{f(b)-f(a)}{b-a}$$

The Attempt at a Solution


I first tried using integration.
$$-1\le f''(x) \le 1$$
integrating from 0 to x,
$$-x\le f'(x)-f'(0) \le x$$
Again integrating from 0 to x,
$$-\frac{x^2}{2}\le f(x)-f(0)-f'(0)x \le \frac{x^2}{2}$$
But even though I got an inequality for f(x), I could not remove the constants.
Then I applied Rolle theorem for f(x). Since f(0)=f(1), there exists a point (at least one point) ##c## such that f'(c)=0.
There exists a point ##X\in [c,x]## such that
$$f''(X)=\frac{f(x)-f(c)}{x-c}$$
Here, ##x\in [c,1]##, and since 1>x>c, x-c<1. Also, ##|f''(x)|\le 1##.
$$f''(X)=\frac{f(x)-f(c)}{x-c}$$
So, ##\frac{f(x)-f(c)}{x-c}\le 1## and ##f(x)-f(c)\le {x-c}##. Hence I get the answer D. Is this correct?
 
Physics news on Phys.org
  • #2
Titan97 said:

Homework Statement


Let ##f## be double differentiable function such that ##|f''(x)|\le 1## for all ##x\in [0,1]##. If f(0)=f(1), then,
A)##|f(x)|>1##
B)##|f(x)|<1##
C)##|f'(x)|>1##
D)##|f'(x)|<1##

Homework Equations


MVT: $$f'(c)=\frac{f(b)-f(a)}{b-a}$$

The Attempt at a Solution


I first tried using integration.

Metahints for real analysis:

If you're given a continuous function on a closed bounded interval and told its values at the end points, consider applying the intermediate value theorem.
If you're given a differentiable function on a closed bounded interval and told its values at the end points, consider applying the mean value theorem.

Then I applied Rolle theorem for f(x). Since f(0)=f(1), there exists a point (at least one point) ##c## such that f'(c)=0.
There exists a point ##X\in [c,x]## such that

$$f''(X)=\frac{f(x)-f(c)}{x-c}$$

There are two problems here. Firstly I take it you are applying the MVT to [itex]f'[/itex] rather than [itex]f[/itex], so [itex]X[/itex] satisfies [tex]
f''(X) = \frac{f'(x) - f'(c)}{x - c} = \frac{f'(x)}{x - c}.[/tex]

Secondly, writing [itex]X \in [c,x][/itex] implicitly requires that [itex]c \leq x[/itex]. But you have to prove a result for every [itex]x \in [0,1][/itex], so you have also to deal with the case [itex]x > c[/itex]. However that requires no further work, since as long as [itex]x \neq c[/itex] there is an [itex]X[/itex] lying between [itex]x[/itex] and [itex]c[/itex] which satisfies the above equation.

Here, ##x\in [c,1]##, and since 1>x>c, x-c<1. Also, ##|f''(x)|\le 1##.

Thus if [itex]x = c[/itex] then [itex]|f'(x)| = 0[/itex], and if [itex]x \neq c[/itex] then [itex]|f'(x)| < \dots[/itex]
 
  • Like
Likes Titan97
  • #3
Titan97 said:

The Attempt at a Solution


I first tried using integration.
$$-1\le f''(x) \le 1$$
integrating from 0 to x,
$$-x\le f'(x)-f'(0) \le x$$
Again integrating from 0 to x,
$$-\frac{x^2}{2}\le f(x)-f(0)-f'(0)x \le \frac{x^2}{2}$$
But even though I got an inequality for f(x), I could not remove the constants.
What does that inequality look like when x = 1? That will give you bounds on f'(0).
 
  • Like
Likes Titan97
  • #4
What if you define ##f'(x) = f'(0) + \int_0^x f''(t) dt ##?
Then using the inequality ## \left| \int f dx \right| \leq \int |f| dx##
You can quickly deduce something about f'(x).

Then, since f(0) = f(1), you can say that
##\int_0^c f'(x) dx = -\int_c^0 f'(x) dx##
You can find a maximum for this based on the same inequality.

I doubt you can say anything about |f(x)| since you aren't given anything that might constrain the initial values. f(0) = f(1) = 100 might be an option based on what you provided.
 
  • #5
@pasmith , that was a typo. I lost internet connection while trying to edit. @RUber, i found the minimum and maximum vaue f'(0) can take. $$-1/2 \le f'(0) \le 1/2$$
 
Last edited:
  • #6
But, from the equation, ##-x\le f'(x)-f(0)\le x## and by substituting the max value of f'(0), ##-x+0.5\le f'(x)\le x+0.5## and at x=1, f'(x)>1. Is that correct?
 
  • #7
Titan97 , do you want a proper solution , or would getting the answer alone simply be enough ?

I haven't actually tried the question , but you could consider a function as f(x) = x(x - 1)/4 .

Solving this can easily tell you about f'(x) ; however I don't believe you can comment on f(x) on the basis of what's given in the original question .

I know this isn't a proper solution , but still ... Hope this helps .
 
  • #8
I want a proper solution. I know many substitutions that can give the answer to such questions.
 
  • #9
Titan97 said:
I want a proper solution. I know many substitutions that can give the answer to such questions.
Okay , first of - You cannot comment on value of f(x) .

For f'(x) - Let f'(x) = k ( f'(0) + x ) , where k is such that -1 <= k <= 1 .

f'(0) = - 0.5 , and so f'(x) = k ( x - 0.5 ) .
( x - 0.5 ) varies from - 0.5 to 0.5 .
⇒ k ( x - 0.5 ) will belong to some interval lesser than equal ( - 0.5 , 0.5 ) - Depending on the value of k

Hope this helps .
 
  • #10
what about post 6?
 
  • #11
Hint: There is a point [itex] x_0\in (0,1)[/itex] such that [itex]f'(x_0)=0[/itex]. If [itex]x[/itex] is another point in [itex](0,1)[/itex], then you can use the fact that [itex]|x-x_0|<1[/itex] together with the information about [itex]f''[/itex] to estimate [itex]f'(x)[/itex].
 
  • #12
Titan97 said:
what about post 6?
F'(0) has a fixed value -
Qwertywerty said:
f'(0) = - 0.5 , and so f'(x) = k ( x - 0.5 ) .
( x - 0.5 ) varies from - 0.5 to 0.5 .
 
  • #13
Qwertywerty said:
Okay , first of - You cannot comment on value of f(x) .

For f'(x) - Let f'(x) = k ( f'(0) + x ) , where k is such that -1 <= k <= 1 .

f'(0) = - 0.5 , and so f'(x) = k ( x - 0.5 ) .
( x - 0.5 ) varies from - 0.5 to 0.5 .
⇒ k ( x - 0.5 ) will belong to some interval lesser than equal ( - 0.5 , 0.5 ) - Depending on the value of k

Hope this helps .
Made a mistake here . I'll get back to you once I correct it .
 
  • #14
RUber said:
What if you define ##f'(x) = f'(0) + \int_0^x f''(t) dt ##?
Then using the inequality ## \left| \int f dx \right| \leq \int |f| dx##
You can quickly deduce something about f'(x).

Then, since f(0) = f(1), you can say that
##\int_0^c f'(x) dx = -\int_c^0 f'(x) dx##
You can find a maximum for this based on the same inequality.

I doubt you can say anything about |f(x)| since you aren't given anything that might constrain the initial values. f(0) = f(1) = 100 might be an option based on what you provided.
If you use the facts here, knowing that f'(c)=0, consider that f' goes directly from its initial value to zero as quickly as possible given constraints on f". The smallest c possible is zero. From there, assume max departure from zero using the max for f". How far from zero can f' get on the remainder of the unit interval?
 
  • #15
I did not understand that properly @RUber .
 
  • #16
I'm not sure why my post #11 is being ignored; it really suffices to solve the problem.
 
  • Like
Likes Titan97
  • #17
Its the same thing i have given in the original post. It does work.
 

Related to Question on Mean Value Theorem

What is the Mean Value Theorem?

The Mean Value Theorem is a fundamental theorem in calculus that states that for any continuous function on a closed interval, there exists a point within that interval where the instantaneous rate of change (derivative) of the function is equal to the average rate of change (slope of the secant line) over the same interval.

What is the significance of the Mean Value Theorem?

The Mean Value Theorem is important because it provides a way to connect the average behavior of a function over an interval to its instantaneous behavior at a specific point. This allows us to make mathematical statements about functions and their derivatives, which are essential tools in many fields of science and engineering.

How is the Mean Value Theorem used in real-world applications?

The Mean Value Theorem has many practical applications, such as in physics where it is used to analyze the motion of objects and in economics where it is used to model the behavior of markets. It is also used in optimization problems, where finding the maximum or minimum of a function is equivalent to finding the point where the derivative is equal to zero.

What are the assumptions of the Mean Value Theorem?

The Mean Value Theorem has two main assumptions: the function must be continuous on a closed interval and differentiable on the open interval. This means that the function must have a defined value at every point within the interval and have a well-defined derivative at every point except possibly the endpoints.

What is the difference between the Mean Value Theorem and the Intermediate Value Theorem?

The Mean Value Theorem and the Intermediate Value Theorem are both fundamental theorems in calculus, but they have different applications. The Mean Value Theorem relates the derivative of a function to its average behavior, while the Intermediate Value Theorem states that if a function is continuous on an interval, it must take on every value between its endpoints. In other words, the Mean Value Theorem deals with rates of change, while the Intermediate Value Theorem deals with the behavior of the function itself.

Similar threads

  • Calculus and Beyond Homework Help
Replies
2
Views
122
  • Calculus and Beyond Homework Help
Replies
12
Views
158
  • Calculus and Beyond Homework Help
Replies
4
Views
721
  • Calculus and Beyond Homework Help
Replies
6
Views
572
  • Calculus and Beyond Homework Help
Replies
21
Views
879
  • Calculus and Beyond Homework Help
Replies
26
Views
2K
  • Calculus and Beyond Homework Help
Replies
11
Views
1K
  • Calculus and Beyond Homework Help
Replies
1
Views
210
  • Calculus and Beyond Homework Help
Replies
15
Views
1K
  • Calculus and Beyond Homework Help
Replies
5
Views
655
Back
Top